Autor Tema: Forma normal de Cantor en $$\omega^\omega$$

0 Usuarios y 1 Visitante están viendo este tema.

01 Mayo, 2023, 05:56 pm
Respuesta #10

franma

  • $$\Large \color{#5b61b3}\pi\,\pi\,\pi\,\pi\,\pi$$
  • Mensajes: 1,433
  • País: uy
  • Karma: +2/-0
  • Sexo: Masculino
Hola Carlos,

Tienes razón. No es por la definición de suma. Es un resultado de aritmética ordinal. En general, se cumple:

(*)   Si \( \alpha \leq \beta \) existe un \( \gamma \) tal que \( \beta = \alpha + \gamma \)

En este caso, como \( \omega^{p_1}\cdot k_1\leq \alpha \), existe un \( \alpha_1 \) tal que \( \alpha = \omega^{p_1}\cdot k_1+\alpha_1 \), y como \( \omega^{p_1}\cdot k_1+\alpha_1<\omega^{p_1}\cdot k_1+\omega^{p_1} \), tiene que ser \( \alpha_1<\omega^{p_1} \).

¿Conoces (*) o hay que probarlo?

Tuve que probarlo para hacer la parte (1) de este mismo ejercicio, aunque tuve algunos problemas con el caso limite, comparto mi prueba:
Si $$\alpha \leq \beta$$ existe un $$\gamma$$ tal que $$\beta = \alpha + \gamma$$
Vamos por induccion en \( \beta \):
-Si \( \beta=0 \) no hay nada que hacer.

-Si \( \beta=\beta'+1 \):
Sea \( \alpha \leq \beta \) entonces o bien \( \alpha=\beta' \) o bien \( \alpha<\beta' \).
  • Si \( \alpha=\beta' \) entonces \( \alpha+1=\beta'+1=\beta \) y tenemos lo buscado.
  • Si \( \alpha<\beta' \) entonces por hipótesis de inducción existe \( \gamma' \) tal que \( \alpha+\gamma'=\beta' \) luego observemos que \( \alpha + (\gamma'+1)=(\alpha+\gamma')+1=\beta \)

-Si \( \beta \) es un ordinal limite:
Sea \( \alpha<\beta \), por hipótesis de inducción dado \( \delta \) tal que \( \alpha \leq \delta <\beta \) existe \( \gamma_\delta \) tal que \( \alpha + \gamma_\delta=\delta \).
Además como \( \beta \) es limite se cumple que \( \displaystyle \beta=\sup_{\delta<\beta}\delta \), entonces:
\( \displaystyle \beta=\sup_{\delta<\beta}\delta=\sup_{\alpha\leq \delta<\beta}\delta=\sup_{\alpha\leq \delta<\beta}(\alpha+\gamma_\delta)=\alpha + \sup_{\alpha\leq \delta<\beta}\gamma_\delta \)
donde la segunda igualdad es porque la sucesión es creciente y el ultimo igual se debe a que dado X un conjunto de ordinales y \( f_\alpha \) una sucesión normal se tiene que \( f_{\sup X}=\sup_{x\in X} f_x \) donde tomamos \( X=\{\gamma_\delta | \alpha\leq\delta<\beta\} \) y la sucesion normal es \( \{\alpha + \beta\}_{\beta:On} \)
[cerrar]

Una vez que tenemos eso, ya tenemos la descomposición \( \alpha=\omega^{p_1}\cdot k_1 +\alpha_1 \) y podemos aplicar la hipótesis de inducción a \( \alpha_1 \).

Luego basta simplificar la expresión utilizando la parte (1) si es necesario ya terminamos, ¿es asi?

Saludos,
Franco.
En ninguna parte puede hallar el hombre un retiro tan apacible y tranquilo como en la intimidad de su alma.

01 Mayo, 2023, 10:49 pm
Respuesta #11

Carlos Ivorra

  • Administrador
  • Mensajes: 11,114
  • País: es
  • Karma: +0/-0
  • Sexo: Masculino
    • Página web personal
Tuve que probarlo para hacer la parte (1) de este mismo ejercicio, aunque tuve algunos problemas con el caso limite, comparto mi prueba:
Si $$\alpha \leq \beta$$ existe un $$\gamma$$ tal que $$\beta = \alpha + \gamma$$
Vamos por induccion en \( \beta \):
-Si \( \beta=0 \) no hay nada que hacer.

-Si \( \beta=\beta'+1 \):
Sea \( \alpha \leq \beta \) entonces o bien \( \alpha=\beta' \) o bien \( \alpha<\beta' \).
  • Si \( \alpha=\beta' \) entonces \( \alpha+1=\beta'+1=\beta \) y tenemos lo buscado.
  • Si \( \alpha<\beta' \) entonces por hipótesis de inducción existe \( \gamma' \) tal que \( \alpha+\gamma'=\beta' \) luego observemos que \( \alpha + (\gamma'+1)=(\alpha+\gamma')+1=\beta \)

-Si \( \beta \) es un ordinal limite:
Sea \( \alpha<\beta \), por hipótesis de inducción dado \( \delta \) tal que \( \alpha \leq \delta <\beta \) existe \( \gamma_\delta \) tal que \( \alpha + \gamma_\delta=\delta \).
Además como \( \beta \) es limite se cumple que \( \displaystyle \beta=\sup_{\delta<\beta}\delta \), entonces:
\( \displaystyle \beta=\sup_{\delta<\beta}\delta=\sup_{\alpha\leq \delta<\beta}\delta=\sup_{\alpha\leq \delta<\beta}(\alpha+\gamma_\delta)=\alpha + \sup_{\alpha\leq \delta<\beta}\gamma_\delta \)
donde la segunda igualdad es porque la sucesión es creciente y el ultimo igual se debe a que dado X un conjunto de ordinales y \( f_\alpha \) una sucesión normal se tiene que \( f_{\sup X}=\sup_{x\in X} f_x \) donde tomamos \( X=\{\gamma_\delta | \alpha\leq\delta<\beta\} \) y la sucesion normal es \( \{\alpha + \beta\}_{\beta:On} \)
[cerrar]

Yo diría que la prueba está bien, pero hay otra algo más breve:

\( \beta \leq \alpha+\beta <\alpha+\beta+1 \), luego existe un ordinal \( \eta \) tal que \( \beta<\alpha+\eta \) (hemos visto que sirve \( \eta = \beta+1 \)), pero tomemos el mínimo \( \eta \) posible. No puede ser \( 0 \), porque \( \alpha \leq \beta \), ni puede ser un ordinal límite, porque en tal caso habría un \( \delta<\eta \) tal que \( \beta <\alpha+\delta \), en contra de la minimalidad, luego tiene que ser \( \eta = \gamma+1 \). De nuevo por la minimalidad, tiene que ser \( \alpha+\gamma \leq \beta <\alpha+\gamma +1 \), luego \( \alpha + \gamma = \beta \).

Una vez que tenemos eso, ya tenemos la descomposición \( \alpha=\omega^{p_1}\cdot k_1 +\alpha_1 \) y podemos aplicar la hipótesis de inducción a \( \alpha_1 \).

Luego basta simplificar la expresión utilizando la parte (1) si es necesario ya terminamos, ¿es asi?

Sí. En realidad no hay nada que simplificar, sino que los exponentes te quedan ya decrecientes, pero cuesta más probarlo que tener en cuenta que, si no lo fueran, se simplificarían por (1).

02 Mayo, 2023, 02:17 am
Respuesta #12

franma

  • $$\Large \color{#5b61b3}\pi\,\pi\,\pi\,\pi\,\pi$$
  • Mensajes: 1,433
  • País: uy
  • Karma: +2/-0
  • Sexo: Masculino
Muchas gracias por toda la ayuda.

Saludos,
Franco.
En ninguna parte puede hallar el hombre un retiro tan apacible y tranquilo como en la intimidad de su alma.